Weekend Sale 70% Discount Offer - Ends in 0d 00h 00m 00s - Coupon code: save70

MCCQE Leak Questions

Page: 4 / 17
Total 230 questions

MCCQE Part 1 Exam Questions and Answers

Question 13

A 53-year-old man with a history of bipolar I disorder is brought to the office by his family. Recently, he has been sleeping for 4 to 5 hours per night, has been fidgety, and is increasinglypreoccupied with his granddaughter's safety. Five days ago, he consulted with your physician colleague and was instructed to exercise and meditate. Last night, he was found running in the street and attempted to hit a relative who was trying to calm him down. His son is dissatisfied with your physician colleague's management. Which one of the following is the most appropriate response?

Options:

A.

Acknowledge your physician colleague's mistake and apologize.

B.

Encourage the son to file a complaint.

C.

Explain that you will now assess the father and that your goal is to treat him.

D.

Point out that exercise and meditation have been proven useful in managing bipolar I disorder.

E.

Share that you would have prescribed a medication after the first assessment.

Question 14

A 35-year-old woman presents to your clinic with double vision and a gritty sensation in her eyes for the past several weeks. On examination, you notice her eyes are bulging. There is inflammation of her conjunctivae and swelling around her eyes. Which one of the following is the most likely diagnosis?

Options:

A.

Orbital pseudotumor

B.

Myasthenia gravis

C.

Allergic conjunctivitis

D.

Periorbital cellulitis

E.

Graves ophthalmopathy

Question 15

A 76-year-old man is brought to the emergency department in an unresponsive state. He has a history of chronic kidney disease with a baseline serum creatinine level of 300 µmol/L (49–93) and a history of dilated cardiomyopathy with an ejection fraction of 30%. On assessment, he has no pulse or blood pressure. Cardiac monitor demonstrates a wide complex tachycardia. Which one of the following recently started medications is the most likely cause of this arrhythmia?

Options:

A.

Spironolactone

B.

Hydrochlorothiazide

C.

Metoprolol

D.

Clopidogrel

E.

Diltiazem

Question 16

A 29-year-old woman presents with vaginal spotting after 6 weeks of amenorrhea. She is asymptomatic otherwise. Serum β-hCG is 2150 IU/L, and pelvic ultrasound shows an empty uterus. She has been trying to conceive for 7 months. Which one of the following is the best next step?

Options:

A.

Repeat pelvic ultrasonography in 10 days.

B.

Perform dilatation and curettage for chorionic villi.

C.

Administer intramuscular methotrexate.

D.

Arrange exploratory laparoscopy.

E.

Repeat serum β-hCG test in 48 hours.

Page: 4 / 17
Total 230 questions